Summation of 'for loop' with conditional? Announcing the arrival of Valued Associate #679:...

Is multiple magic items in one inherently imbalanced?

Tannaka duality for semisimple groups

How can I prevent/balance waiting and turtling as a response to cooldown mechanics

The Nth Gryphon Number

What order were files/directories output in dir?

Relating to the President and obstruction, were Mueller's conclusions preordained?

Central Vacuuming: Is it worth it, and how does it compare to normal vacuuming?

Why datecode is SO IMPORTANT to chip manufacturers?

Why shouldn't this prove the Prime Number Theorem?

How can god fight other gods?

Did Mueller's report provide an evidentiary basis for the claim of Russian govt election interference via social media?

Is it dangerous to install hacking tools on my private linux machine?

Most effective melee weapons for arboreal combat? (pre-gunpowder technology)

Constant factor of an array

I got rid of Mac OSX and replaced it with linux but now I can't change it back to OSX or windows

Why is a lens darker than other ones when applying the same settings?

How to ask rejected full-time candidates to apply to teach individual courses?

Where is the Next Backup Size entry on iOS 12?

Co-worker has annoying ringtone

Nose gear failure in single prop aircraft: belly landing or nose-gear up landing?

What is a more techy Technical Writer job title that isn't cutesy or confusing?

What does 丫 mean? 丫是什么意思?

What does it mean that physics no longer uses mechanical models to describe phenomena?

How much damage would a cupful of neutron star matter do to the Earth?



Summation of 'for loop' with conditional?



Announcing the arrival of Valued Associate #679: Cesar Manara
Planned maintenance scheduled April 23, 2019 at 23:30 UTC (7:30pm US/Eastern)Proof for a summation-procedure using the matrix of Eulerian numbers?Solving Summation ExpressionsHelp me understand this algorithm problem.Finding a recurrence for a sumSimplifying $sum_{i=1}^{n-2}i(n-1-i)$Write the following for loop as a double summationSummation with arithmetic seriesCalculating part of a summation function as a constant?Is summation the equivalent of a for loop? (Programming)Conditional summation with or without complex roots












1












$begingroup$


I am trying to convert instances of nested 'for' loops into a summation expression. The current code fragment I have is:



for i = 1 to n:
for j = 1 to n:
if (i*j >= n):
for k = 1 to n:
sum++
endif


Basically, the 'if' conditional is confusing me. I know that the loops prior will be called n^2 times, but the third loop is only called when $i*j >= n$. How would I write the third summation to account for this, and then evaluate the overall loop's time complexity?










share|cite|improve this question









$endgroup$












  • $begingroup$
    This is often context dependent. If, say, you wanted to iterate over the entries $a_{ij}$ of an $n times n$ matrix, then the first two loops in your code sample could be adequately expressed as $$sum_{ij geq n} a_{ij}$$. Perhaps if you provided more context about why you have this problem we will be able to provide more helpful answers.
    $endgroup$
    – Brian
    Mar 26 at 1:18










  • $begingroup$
    You want to convert the nested loops into a summation, but then you say you want to evaluate the time complexity of your code, which does not require you to convert the loops into a summation, and, in fact, does not require you to find what value sum has after the loops.
    $endgroup$
    – Fabio Somenzi
    Mar 26 at 3:15
















1












$begingroup$


I am trying to convert instances of nested 'for' loops into a summation expression. The current code fragment I have is:



for i = 1 to n:
for j = 1 to n:
if (i*j >= n):
for k = 1 to n:
sum++
endif


Basically, the 'if' conditional is confusing me. I know that the loops prior will be called n^2 times, but the third loop is only called when $i*j >= n$. How would I write the third summation to account for this, and then evaluate the overall loop's time complexity?










share|cite|improve this question









$endgroup$












  • $begingroup$
    This is often context dependent. If, say, you wanted to iterate over the entries $a_{ij}$ of an $n times n$ matrix, then the first two loops in your code sample could be adequately expressed as $$sum_{ij geq n} a_{ij}$$. Perhaps if you provided more context about why you have this problem we will be able to provide more helpful answers.
    $endgroup$
    – Brian
    Mar 26 at 1:18










  • $begingroup$
    You want to convert the nested loops into a summation, but then you say you want to evaluate the time complexity of your code, which does not require you to convert the loops into a summation, and, in fact, does not require you to find what value sum has after the loops.
    $endgroup$
    – Fabio Somenzi
    Mar 26 at 3:15














1












1








1


1



$begingroup$


I am trying to convert instances of nested 'for' loops into a summation expression. The current code fragment I have is:



for i = 1 to n:
for j = 1 to n:
if (i*j >= n):
for k = 1 to n:
sum++
endif


Basically, the 'if' conditional is confusing me. I know that the loops prior will be called n^2 times, but the third loop is only called when $i*j >= n$. How would I write the third summation to account for this, and then evaluate the overall loop's time complexity?










share|cite|improve this question









$endgroup$




I am trying to convert instances of nested 'for' loops into a summation expression. The current code fragment I have is:



for i = 1 to n:
for j = 1 to n:
if (i*j >= n):
for k = 1 to n:
sum++
endif


Basically, the 'if' conditional is confusing me. I know that the loops prior will be called n^2 times, but the third loop is only called when $i*j >= n$. How would I write the third summation to account for this, and then evaluate the overall loop's time complexity?







summation






share|cite|improve this question













share|cite|improve this question











share|cite|improve this question




share|cite|improve this question










asked Mar 26 at 0:59









bpryanbpryan

83




83












  • $begingroup$
    This is often context dependent. If, say, you wanted to iterate over the entries $a_{ij}$ of an $n times n$ matrix, then the first two loops in your code sample could be adequately expressed as $$sum_{ij geq n} a_{ij}$$. Perhaps if you provided more context about why you have this problem we will be able to provide more helpful answers.
    $endgroup$
    – Brian
    Mar 26 at 1:18










  • $begingroup$
    You want to convert the nested loops into a summation, but then you say you want to evaluate the time complexity of your code, which does not require you to convert the loops into a summation, and, in fact, does not require you to find what value sum has after the loops.
    $endgroup$
    – Fabio Somenzi
    Mar 26 at 3:15


















  • $begingroup$
    This is often context dependent. If, say, you wanted to iterate over the entries $a_{ij}$ of an $n times n$ matrix, then the first two loops in your code sample could be adequately expressed as $$sum_{ij geq n} a_{ij}$$. Perhaps if you provided more context about why you have this problem we will be able to provide more helpful answers.
    $endgroup$
    – Brian
    Mar 26 at 1:18










  • $begingroup$
    You want to convert the nested loops into a summation, but then you say you want to evaluate the time complexity of your code, which does not require you to convert the loops into a summation, and, in fact, does not require you to find what value sum has after the loops.
    $endgroup$
    – Fabio Somenzi
    Mar 26 at 3:15
















$begingroup$
This is often context dependent. If, say, you wanted to iterate over the entries $a_{ij}$ of an $n times n$ matrix, then the first two loops in your code sample could be adequately expressed as $$sum_{ij geq n} a_{ij}$$. Perhaps if you provided more context about why you have this problem we will be able to provide more helpful answers.
$endgroup$
– Brian
Mar 26 at 1:18




$begingroup$
This is often context dependent. If, say, you wanted to iterate over the entries $a_{ij}$ of an $n times n$ matrix, then the first two loops in your code sample could be adequately expressed as $$sum_{ij geq n} a_{ij}$$. Perhaps if you provided more context about why you have this problem we will be able to provide more helpful answers.
$endgroup$
– Brian
Mar 26 at 1:18












$begingroup$
You want to convert the nested loops into a summation, but then you say you want to evaluate the time complexity of your code, which does not require you to convert the loops into a summation, and, in fact, does not require you to find what value sum has after the loops.
$endgroup$
– Fabio Somenzi
Mar 26 at 3:15




$begingroup$
You want to convert the nested loops into a summation, but then you say you want to evaluate the time complexity of your code, which does not require you to convert the loops into a summation, and, in fact, does not require you to find what value sum has after the loops.
$endgroup$
– Fabio Somenzi
Mar 26 at 3:15










3 Answers
3






active

oldest

votes


















1












$begingroup$

Hint:



The "for" cycle in $k$ is very easy to turn into something simpler...



As for the other parts, see if you can split the problem into easier steps. For example, what happens for $i=1$? And what happens for $i=2$? And $i=3$? And...






share|cite|improve this answer









$endgroup$





















    0












    $begingroup$


    We obtain
    begin{align*}
    color{blue}{sum_{i=1}^n}&color{blue}{sum_{j=1}^n[i jgeq n]sum_{k=1}^n1}tag{1}\
    &=nsum_{i=1}^nsum_{j=1}^n[i jgeq n]\
    &=nleft(sum_{i=1}^nsum_{{j=leftlfloor n/i rightrfloor}atop{imid n}}^n1+sum_{i=1}^nsum_{{j=leftlfloor n/i rightrfloor+1}atop{inot mid n}}^n1right)tag{2}\
    &=nleft(sum_{i=1}^nsum_{j=leftlfloor n/i rightrfloor+1}^n1+sum_{{i=1}atop{imid n}}^n1right)\
    &=nleft(sum_{i=1}^nleft(n-leftlfloorfrac{n}{i}rightrfloorright)+tau(n)right)tag{3}\
    &=nleft(n^2-sum_{i=1}^nleftlfloorfrac{n}{i}rightrfloor+tau(n)right)\
    &,,color{blue}{=nleft(n^2-sum_{i=1}^{n-1}tau(i)right)}tag{4}\
    &,,color{blue}{sim n^3-n^2left(log n+2gamma-1right)+Oleft(n^{3/2}right)}tag{5}
    end{align*}




    Comment:




    • In (1) we use Iverson brackets to represent the condition $ijgeq n$.


    • In (2) we rewrite the expression using the floor function. Note the $pm 1$ technicality regarding the range of summation.


    • In (3) we simplify the inner sum of the left-hand term and use the $tau$-function which counts the number of divisors.


    • In (4) we use an identity stated in A006218 to get rid of the floor function.


    • In (5) we use an asymptotic expansion which is also stated in OEIS/A006218.







    share|cite|improve this answer









    $endgroup$





















      0












      $begingroup$

      At the basic level, for loops are also rewritable as conditionals. Anyways, on to the meat of the problem. The conditional can be removed if you replace 1 in the previous loop with ceil(n/i). So it can be rewritten as:



      for i = 1 to n:
      for j = ceil(n/i) to n:
      for k = 1 to n:
      sum++



      Then the inner 2 loops compress to:



      sum+=n*(n-ceil(n/i)+1)



      Which then gets called n times, but the sum can be wriiten as:



      $$nsum_{i=1}^{n}(n-lceilfrac{n}{i}rceil+1)$$



      As to the time complexity of the code as wriiten..., it'll take, okay I don't quite know that part.






      share|cite|improve this answer









      $endgroup$














        Your Answer








        StackExchange.ready(function() {
        var channelOptions = {
        tags: "".split(" "),
        id: "69"
        };
        initTagRenderer("".split(" "), "".split(" "), channelOptions);

        StackExchange.using("externalEditor", function() {
        // Have to fire editor after snippets, if snippets enabled
        if (StackExchange.settings.snippets.snippetsEnabled) {
        StackExchange.using("snippets", function() {
        createEditor();
        });
        }
        else {
        createEditor();
        }
        });

        function createEditor() {
        StackExchange.prepareEditor({
        heartbeatType: 'answer',
        autoActivateHeartbeat: false,
        convertImagesToLinks: true,
        noModals: true,
        showLowRepImageUploadWarning: true,
        reputationToPostImages: 10,
        bindNavPrevention: true,
        postfix: "",
        imageUploader: {
        brandingHtml: "Powered by u003ca class="icon-imgur-white" href="https://imgur.com/"u003eu003c/au003e",
        contentPolicyHtml: "User contributions licensed under u003ca href="https://creativecommons.org/licenses/by-sa/3.0/"u003ecc by-sa 3.0 with attribution requiredu003c/au003e u003ca href="https://stackoverflow.com/legal/content-policy"u003e(content policy)u003c/au003e",
        allowUrls: true
        },
        noCode: true, onDemand: true,
        discardSelector: ".discard-answer"
        ,immediatelyShowMarkdownHelp:true
        });


        }
        });














        draft saved

        draft discarded


















        StackExchange.ready(
        function () {
        StackExchange.openid.initPostLogin('.new-post-login', 'https%3a%2f%2fmath.stackexchange.com%2fquestions%2f3162547%2fsummation-of-for-loop-with-conditional%23new-answer', 'question_page');
        }
        );

        Post as a guest















        Required, but never shown

























        3 Answers
        3






        active

        oldest

        votes








        3 Answers
        3






        active

        oldest

        votes









        active

        oldest

        votes






        active

        oldest

        votes









        1












        $begingroup$

        Hint:



        The "for" cycle in $k$ is very easy to turn into something simpler...



        As for the other parts, see if you can split the problem into easier steps. For example, what happens for $i=1$? And what happens for $i=2$? And $i=3$? And...






        share|cite|improve this answer









        $endgroup$


















          1












          $begingroup$

          Hint:



          The "for" cycle in $k$ is very easy to turn into something simpler...



          As for the other parts, see if you can split the problem into easier steps. For example, what happens for $i=1$? And what happens for $i=2$? And $i=3$? And...






          share|cite|improve this answer









          $endgroup$
















            1












            1








            1





            $begingroup$

            Hint:



            The "for" cycle in $k$ is very easy to turn into something simpler...



            As for the other parts, see if you can split the problem into easier steps. For example, what happens for $i=1$? And what happens for $i=2$? And $i=3$? And...






            share|cite|improve this answer









            $endgroup$



            Hint:



            The "for" cycle in $k$ is very easy to turn into something simpler...



            As for the other parts, see if you can split the problem into easier steps. For example, what happens for $i=1$? And what happens for $i=2$? And $i=3$? And...







            share|cite|improve this answer












            share|cite|improve this answer



            share|cite|improve this answer










            answered Mar 26 at 1:18









            ErtxiemErtxiem

            941212




            941212























                0












                $begingroup$


                We obtain
                begin{align*}
                color{blue}{sum_{i=1}^n}&color{blue}{sum_{j=1}^n[i jgeq n]sum_{k=1}^n1}tag{1}\
                &=nsum_{i=1}^nsum_{j=1}^n[i jgeq n]\
                &=nleft(sum_{i=1}^nsum_{{j=leftlfloor n/i rightrfloor}atop{imid n}}^n1+sum_{i=1}^nsum_{{j=leftlfloor n/i rightrfloor+1}atop{inot mid n}}^n1right)tag{2}\
                &=nleft(sum_{i=1}^nsum_{j=leftlfloor n/i rightrfloor+1}^n1+sum_{{i=1}atop{imid n}}^n1right)\
                &=nleft(sum_{i=1}^nleft(n-leftlfloorfrac{n}{i}rightrfloorright)+tau(n)right)tag{3}\
                &=nleft(n^2-sum_{i=1}^nleftlfloorfrac{n}{i}rightrfloor+tau(n)right)\
                &,,color{blue}{=nleft(n^2-sum_{i=1}^{n-1}tau(i)right)}tag{4}\
                &,,color{blue}{sim n^3-n^2left(log n+2gamma-1right)+Oleft(n^{3/2}right)}tag{5}
                end{align*}




                Comment:




                • In (1) we use Iverson brackets to represent the condition $ijgeq n$.


                • In (2) we rewrite the expression using the floor function. Note the $pm 1$ technicality regarding the range of summation.


                • In (3) we simplify the inner sum of the left-hand term and use the $tau$-function which counts the number of divisors.


                • In (4) we use an identity stated in A006218 to get rid of the floor function.


                • In (5) we use an asymptotic expansion which is also stated in OEIS/A006218.







                share|cite|improve this answer









                $endgroup$


















                  0












                  $begingroup$


                  We obtain
                  begin{align*}
                  color{blue}{sum_{i=1}^n}&color{blue}{sum_{j=1}^n[i jgeq n]sum_{k=1}^n1}tag{1}\
                  &=nsum_{i=1}^nsum_{j=1}^n[i jgeq n]\
                  &=nleft(sum_{i=1}^nsum_{{j=leftlfloor n/i rightrfloor}atop{imid n}}^n1+sum_{i=1}^nsum_{{j=leftlfloor n/i rightrfloor+1}atop{inot mid n}}^n1right)tag{2}\
                  &=nleft(sum_{i=1}^nsum_{j=leftlfloor n/i rightrfloor+1}^n1+sum_{{i=1}atop{imid n}}^n1right)\
                  &=nleft(sum_{i=1}^nleft(n-leftlfloorfrac{n}{i}rightrfloorright)+tau(n)right)tag{3}\
                  &=nleft(n^2-sum_{i=1}^nleftlfloorfrac{n}{i}rightrfloor+tau(n)right)\
                  &,,color{blue}{=nleft(n^2-sum_{i=1}^{n-1}tau(i)right)}tag{4}\
                  &,,color{blue}{sim n^3-n^2left(log n+2gamma-1right)+Oleft(n^{3/2}right)}tag{5}
                  end{align*}




                  Comment:




                  • In (1) we use Iverson brackets to represent the condition $ijgeq n$.


                  • In (2) we rewrite the expression using the floor function. Note the $pm 1$ technicality regarding the range of summation.


                  • In (3) we simplify the inner sum of the left-hand term and use the $tau$-function which counts the number of divisors.


                  • In (4) we use an identity stated in A006218 to get rid of the floor function.


                  • In (5) we use an asymptotic expansion which is also stated in OEIS/A006218.







                  share|cite|improve this answer









                  $endgroup$
















                    0












                    0








                    0





                    $begingroup$


                    We obtain
                    begin{align*}
                    color{blue}{sum_{i=1}^n}&color{blue}{sum_{j=1}^n[i jgeq n]sum_{k=1}^n1}tag{1}\
                    &=nsum_{i=1}^nsum_{j=1}^n[i jgeq n]\
                    &=nleft(sum_{i=1}^nsum_{{j=leftlfloor n/i rightrfloor}atop{imid n}}^n1+sum_{i=1}^nsum_{{j=leftlfloor n/i rightrfloor+1}atop{inot mid n}}^n1right)tag{2}\
                    &=nleft(sum_{i=1}^nsum_{j=leftlfloor n/i rightrfloor+1}^n1+sum_{{i=1}atop{imid n}}^n1right)\
                    &=nleft(sum_{i=1}^nleft(n-leftlfloorfrac{n}{i}rightrfloorright)+tau(n)right)tag{3}\
                    &=nleft(n^2-sum_{i=1}^nleftlfloorfrac{n}{i}rightrfloor+tau(n)right)\
                    &,,color{blue}{=nleft(n^2-sum_{i=1}^{n-1}tau(i)right)}tag{4}\
                    &,,color{blue}{sim n^3-n^2left(log n+2gamma-1right)+Oleft(n^{3/2}right)}tag{5}
                    end{align*}




                    Comment:




                    • In (1) we use Iverson brackets to represent the condition $ijgeq n$.


                    • In (2) we rewrite the expression using the floor function. Note the $pm 1$ technicality regarding the range of summation.


                    • In (3) we simplify the inner sum of the left-hand term and use the $tau$-function which counts the number of divisors.


                    • In (4) we use an identity stated in A006218 to get rid of the floor function.


                    • In (5) we use an asymptotic expansion which is also stated in OEIS/A006218.







                    share|cite|improve this answer









                    $endgroup$




                    We obtain
                    begin{align*}
                    color{blue}{sum_{i=1}^n}&color{blue}{sum_{j=1}^n[i jgeq n]sum_{k=1}^n1}tag{1}\
                    &=nsum_{i=1}^nsum_{j=1}^n[i jgeq n]\
                    &=nleft(sum_{i=1}^nsum_{{j=leftlfloor n/i rightrfloor}atop{imid n}}^n1+sum_{i=1}^nsum_{{j=leftlfloor n/i rightrfloor+1}atop{inot mid n}}^n1right)tag{2}\
                    &=nleft(sum_{i=1}^nsum_{j=leftlfloor n/i rightrfloor+1}^n1+sum_{{i=1}atop{imid n}}^n1right)\
                    &=nleft(sum_{i=1}^nleft(n-leftlfloorfrac{n}{i}rightrfloorright)+tau(n)right)tag{3}\
                    &=nleft(n^2-sum_{i=1}^nleftlfloorfrac{n}{i}rightrfloor+tau(n)right)\
                    &,,color{blue}{=nleft(n^2-sum_{i=1}^{n-1}tau(i)right)}tag{4}\
                    &,,color{blue}{sim n^3-n^2left(log n+2gamma-1right)+Oleft(n^{3/2}right)}tag{5}
                    end{align*}




                    Comment:




                    • In (1) we use Iverson brackets to represent the condition $ijgeq n$.


                    • In (2) we rewrite the expression using the floor function. Note the $pm 1$ technicality regarding the range of summation.


                    • In (3) we simplify the inner sum of the left-hand term and use the $tau$-function which counts the number of divisors.


                    • In (4) we use an identity stated in A006218 to get rid of the floor function.


                    • In (5) we use an asymptotic expansion which is also stated in OEIS/A006218.








                    share|cite|improve this answer












                    share|cite|improve this answer



                    share|cite|improve this answer










                    answered Mar 26 at 12:58









                    Markus ScheuerMarkus Scheuer

                    64.7k460154




                    64.7k460154























                        0












                        $begingroup$

                        At the basic level, for loops are also rewritable as conditionals. Anyways, on to the meat of the problem. The conditional can be removed if you replace 1 in the previous loop with ceil(n/i). So it can be rewritten as:



                        for i = 1 to n:
                        for j = ceil(n/i) to n:
                        for k = 1 to n:
                        sum++



                        Then the inner 2 loops compress to:



                        sum+=n*(n-ceil(n/i)+1)



                        Which then gets called n times, but the sum can be wriiten as:



                        $$nsum_{i=1}^{n}(n-lceilfrac{n}{i}rceil+1)$$



                        As to the time complexity of the code as wriiten..., it'll take, okay I don't quite know that part.






                        share|cite|improve this answer









                        $endgroup$


















                          0












                          $begingroup$

                          At the basic level, for loops are also rewritable as conditionals. Anyways, on to the meat of the problem. The conditional can be removed if you replace 1 in the previous loop with ceil(n/i). So it can be rewritten as:



                          for i = 1 to n:
                          for j = ceil(n/i) to n:
                          for k = 1 to n:
                          sum++



                          Then the inner 2 loops compress to:



                          sum+=n*(n-ceil(n/i)+1)



                          Which then gets called n times, but the sum can be wriiten as:



                          $$nsum_{i=1}^{n}(n-lceilfrac{n}{i}rceil+1)$$



                          As to the time complexity of the code as wriiten..., it'll take, okay I don't quite know that part.






                          share|cite|improve this answer









                          $endgroup$
















                            0












                            0








                            0





                            $begingroup$

                            At the basic level, for loops are also rewritable as conditionals. Anyways, on to the meat of the problem. The conditional can be removed if you replace 1 in the previous loop with ceil(n/i). So it can be rewritten as:



                            for i = 1 to n:
                            for j = ceil(n/i) to n:
                            for k = 1 to n:
                            sum++



                            Then the inner 2 loops compress to:



                            sum+=n*(n-ceil(n/i)+1)



                            Which then gets called n times, but the sum can be wriiten as:



                            $$nsum_{i=1}^{n}(n-lceilfrac{n}{i}rceil+1)$$



                            As to the time complexity of the code as wriiten..., it'll take, okay I don't quite know that part.






                            share|cite|improve this answer









                            $endgroup$



                            At the basic level, for loops are also rewritable as conditionals. Anyways, on to the meat of the problem. The conditional can be removed if you replace 1 in the previous loop with ceil(n/i). So it can be rewritten as:



                            for i = 1 to n:
                            for j = ceil(n/i) to n:
                            for k = 1 to n:
                            sum++



                            Then the inner 2 loops compress to:



                            sum+=n*(n-ceil(n/i)+1)



                            Which then gets called n times, but the sum can be wriiten as:



                            $$nsum_{i=1}^{n}(n-lceilfrac{n}{i}rceil+1)$$



                            As to the time complexity of the code as wriiten..., it'll take, okay I don't quite know that part.







                            share|cite|improve this answer












                            share|cite|improve this answer



                            share|cite|improve this answer










                            answered Mar 27 at 16:25









                            Roddy MacPheeRoddy MacPhee

                            927118




                            927118






























                                draft saved

                                draft discarded




















































                                Thanks for contributing an answer to Mathematics Stack Exchange!


                                • Please be sure to answer the question. Provide details and share your research!

                                But avoid



                                • Asking for help, clarification, or responding to other answers.

                                • Making statements based on opinion; back them up with references or personal experience.


                                Use MathJax to format equations. MathJax reference.


                                To learn more, see our tips on writing great answers.




                                draft saved


                                draft discarded














                                StackExchange.ready(
                                function () {
                                StackExchange.openid.initPostLogin('.new-post-login', 'https%3a%2f%2fmath.stackexchange.com%2fquestions%2f3162547%2fsummation-of-for-loop-with-conditional%23new-answer', 'question_page');
                                }
                                );

                                Post as a guest















                                Required, but never shown





















































                                Required, but never shown














                                Required, but never shown












                                Required, but never shown







                                Required, but never shown

































                                Required, but never shown














                                Required, but never shown












                                Required, but never shown







                                Required, but never shown







                                Popular posts from this blog

                                Magento 2 - Add success message with knockout Planned maintenance scheduled April 23, 2019 at 23:30 UTC (7:30pm US/Eastern) Announcing the arrival of Valued Associate #679: Cesar Manara Unicorn Meta Zoo #1: Why another podcast?Success / Error message on ajax request$.widget is not a function when loading a homepage after add custom jQuery on custom themeHow can bind jQuery to current document in Magento 2 When template load by ajaxRedirect page using plugin in Magento 2Magento 2 - Update quantity and totals of cart page without page reload?Magento 2: Quote data not loaded on knockout checkoutMagento 2 : I need to change add to cart success message after adding product into cart through pluginMagento 2.2.5 How to add additional products to cart from new checkout step?Magento 2 Add error/success message with knockoutCan't validate Post Code on checkout page

                                Fil:Tokke komm.svg

                                Where did Arya get these scars? Unicorn Meta Zoo #1: Why another podcast? Announcing the arrival of Valued Associate #679: Cesar Manara Favourite questions and answers from the 1st quarter of 2019Why did Arya refuse to end it?Has the pronunciation of Arya Stark's name changed?Has Arya forgiven people?Why did Arya Stark lose her vision?Why can Arya still use the faces?Has the Narrow Sea become narrower?Does Arya Stark know how to make poisons outside of the House of Black and White?Why did Nymeria leave Arya?Why did Arya not kill the Lannister soldiers she encountered in the Riverlands?What is the current canonical age of Sansa, Bran and Arya Stark?